P/S UWorld Missed Qs

Réussis tes devoirs et examens dès maintenant avec Quizwiz!

Schacter-Singer theory

the two-factor theory, states that our emotional experiences are caused by the cognitive interpretation of our body's physiological responses to the specific situation we are in.

An HP group peer mentor would have been most likely to have participants focus on: A) accepting the circumstances of their child's condition and pursuing personal growth. B) replacing negative, maladaptive thoughts with positive, adaptive thoughts. C) uncovering unconscious conflicts and feelings about their child's condition. D) treating underlying physiological causes of their depression before engaging in therapy.

A - Humanistic psychology takes an optimistic approach, emphasizing the positive and creative aspects of human nature. It asserts that individuals strive toward self-actualization (fullest potential) and personal growth.

Which brain waves are present in the least amounts during sleep in the elderly population compared to the younger population? A) Alpha waves B) Beta waves C) Theta waves D) Delta waves

B - REM sleep consists of beta waves (similar to an awake state), and elderly people spend less time in REM sleep than younger people. As a result, elderly people will show fewer beta waves during sleep (choice B is correct). Alpha waves are present in an awake but drowsy state (choice A is incorrect). Theta waves are present in light sleep (choice C is incorrect). Delta waves are present in deep sleep (choice D is incorrect).

In response to a stressor, the sympathetic nervous system will cause:

- Inhibition of saliva production - Dilation of the pupils: to take in as much light as possible to better assess the dangerous situation - Accelerated heart rate: to transport oxygen to the muscles - Slowed digestion: so blood can be diverted to the muscles - Airway dilation: to provide more oxygen with each inhalation - Increased glycogenolysis in the liver and increased blood glucose levels: to provide energy

According to Mead, which behavior is an expression of the Me component of the Self? A) An individual studying for an exam instead of going to a party B) An individual skipping work because they want to go to a concert C) An individual ignoring the emotional needs of a significant other D) An individual dressing inappropriately for a job interview

A

Kelly's covariation model outline three types of cues are used to assist in making attributions. Which of the following terms is not one of these three categories? A) Familiarity cues. B) Distinctiveness cues. C) Consensus cues. D) Consistency cues.

A - Kelly's cues include distinctiveness (how likely the person is to behave this way in different situation), consensus (how many other individuals typically exhibit this behavior or how accepted it is in society), and consistency (how similar the person's behavior is over time). Familiarity is not one of these types of queue.

Researchers hypothesized that cocaine is pharmacologically active in the primate fetal brain when pregnant primates are administered cocaine at doses typically used by human drug users. Which experimental observation best supports their hypothesis? A) Glucose metabolism in the fetal brain increased. B) Imaging studies showed that cocaine entered the fetal circulation. C) The fetus had an increase in tolerance to pain. D) Imaging studies showed increased internalization of dopamine receptors

A - The answer to this question is A because cocaine is a stimulant it would have a physiological effect similar to stress and hence glucose metabolism is expected to increase. B is incorrect because transport of cocaine to the fetal circulation does not indicate whether the drug is pharmacologically active in the fetus. C is incorrect because the fetus may not be mature enough to feel pain. In any case, pain relief is associated with heroin, not cocaine. D is incorrect because, if the receptors were internalized, dopamine could not be pharmacologically active. Neurotransmitters bind to receptors on the outside, not inside, of neurons.

A patient with Korsakoff syndrome is very guarded toward the therapist and is reluctant to answer the therapist's questions. At subsequent appointments, the patient indicates not recognizing the therapist but is less guarded and more willing to answer questions. This suggests that the patient: A) Has no explicit memory of meeting the therapist but does have an implicit memory of the meeting. B) Has no episodic memory of meeting the therapist but does have a semantic memory of the meeting. C) Has no short-term memory of meeting the therapist but does have a long-term memory of the meeting. D) Has no sensory memory of meeting the therapist but does have a procedural memory of the meeting.

A - The answer to this question is A because the patient does not remember the therapist explicitly, but the change in his behavior indicates that he has an implicit memory of therapist, which likely results in him becoming less guarded.

Interpretation of intelligence test scores is based on the assumption that the scores are normally distributed within a population such that: A) More than two-thirds of children will score between 85 and 115. B) A quarter of all children will perform poorly on least one component of the exam. C) Significantly more children will score at the extremes of the scoring distribution than in the middle. D) Only exceptionally gifted children would have IQ scores above 85 by age 6; perfect test scores are rare during early childhood.

A - The answer to this question is A because, based on the standardization system used to score IQ, the Wechsler Scales of Intelligence (WISC) scores are "normalized" to a mean of 100 and a standard deviation of 15. So 68% of the scores will be between 85 and 115.

Based on the information in the final paragraph, a successful strategy for promoting weight loss would be to counteract which cognitive bias? A) Confirmation B) Self-serving C) Actor-observer D) Hindsight

A- According to the passage, overweight Americans may not realize they are overweight due to changing social norms about acceptable body size. As others (friends, coworkers) gain weight, being overweight tends to look increasingly normal, making it easier to believe that being too heavy is merely average. In other words, overweight Americans might look only at evidence confirming their belief that their own weight is normal, ignoring evidence that it is not. Therefore, a successful strategy for promoting weight loss would be to counteract confirmation bias.

To assess the relationship between chronic disease and spatial inequality in a given area, researchers systematically observe and interview individuals in their communities. This is best described as which of the following? A) Experimental design B) Ethnographic study C) Social network analysis D) Content analysis

B - Ethnography is a qualitative method for the scientific study of human social phenomena. Using observation and interviews, ethnographies study people in their natural environments (within their own communities) and provide descriptive information about the cultures, behaviors, norms, and values in a given geographic location.

A researcher compares the brain sizes of two groups of rats. Group 1 has been raised in impoverished cages with minimal access to socialization and exercise. Group 2 has been raised in enriched cages with regular access to socialization and exercise. This is best described as a study of: A) Long-term potentiation. B) Neural plasticity. C) Hemispheric lateralization. D) Synaptic reuptake.

B - The answer to this question is B because changes in brain size as a function of environmental influences is an example of neural plasticity.

The researchers determined that two participants in the S group were taking medication that may have exacerbated their negative symptoms. Which classification of medication were these participants most likely taking? A) Atypical antipsychotics B) Neuroleptics C) Hallucinogens D) Stimulants

B - The answer to this question is B because neuroleptics are the first antipsychotic drugs used to treat schizophrenia and though they are effective in treating positive symptoms, their side effects include cognitive dulling, which can exacerbate negative symptoms.

An individual experiences shortness of breath and panic in confined spaces. To overcome this reaction, the individual decides to spend time in a confined space until the shortness of breath and panic response are eliminated. The patient is using which method to eliminate the symptoms? A) Extinction of an operantly conditioned response B) Extinction of a classically conditioned response C) Counter conditioning D) Aversive conditioning

B - The answer to this question is B because phobic responses are usually acquired through classical conditioning. Exposing oneself to the conditioned stimulus (in this case, the confined space) until the conditioned phobia is no longer elicited is consistent with the extinction of a classically conditioned response.

A researcher is interested in assessing the size of a patient's brain ventricles. Which method(s) is(are) appropriate for this purpose? I. An EEG II. A CAT scan III. A PET scan A) I only B) II only C) I and III only D) II and III only

B - The answer to this question is B because the researcher is interested in the structure of the brain, which can be studied through a CAT scan.

Cognitive-behavioral therapy (CBT) would be most likely to address PTSD symptoms through: A) A person-centered approach focused on empathy. B) Systematic desensitization of traumatic triggers or memories. C) Uncovering unconscious memories associated with the trauma. D) Examination of the initial cause of the trauma.

B- Cognitive-behavioral therapy (CBT) is an effective treatment approach for trauma-related disorders (eg, PTSD) and anxiety disorders (eg, GAD). CBT focuses on problematic symptoms like maladaptive thoughts (eg, persistent negative beliefs about oneself or the world) and destructive/unproductive behaviors (eg, drinking, avoiding social situations). The goal of CBT is to replace maladaptive thoughts and behaviors with ones that are healthy and positive. One CBT technique involves the use of systematic desensitization to gradually and systematically expose an individual with PTSD to the feared stimulus (eg, traumatic memory) while actively modifying behavior using coping or relaxation techniques (eg, deep breathing). For example, a therapist could ask a patient to recall aspects of the trauma. Deep breathing would be used to help the patient cope with their distress. Over time, the therapist and patient would repeat this process to confront more detail of the traumatic event. When the patient's reaction to the stimulus (ie, the memory of the trauma) lessens, desensitization has occurred.

A person looking at the night sky notices she no longer sees a dim star when her gaze remains fixated on it, but the dim star reappears when she shifts her gaze to one side of the star. What is the most likely cause of the reappearance of the dim star in her vision? Shifting her gaze moved the image of the dim star: A) Away from the blind spot where no photoreceptors are present. B) To a region of the retina where photoreceptors have a higher threshold for light detection. C) Away from the fovea toward the periphery of the retina. D) To a region of the eye where photoreceptors contain more than one type of retinal pigment.

C - The answer to this question is C because the fovea is the part of the retina that contains a high density of cones for daytime vision, whereas, the periphery of the retina contains a high density of rods which are more photosensitive and can detect dim light.

If a family views an evening meal as having specific consistent compounds, but a hired nutritionist works to improve the family's diet by separating them from those expectations, the nutritionist's approach is utilizing: A) Observational modeling B) Operant conditioning C) Social constructivism D) Classical conditioning

C- The nutritionist is working to change behavior by altering something that was constructed as an absolute in order to correct the problem. Social constructivism is the family's view of dinner is constructed and accepted, and the nutritionist is attempting to break down the construction.

A patient experiences a debilitating depressive episode after the loss of a job. A psychologist suggests that the patient had a genetic predisposition to depression and that unemployment acted as a trigger. The psychologist is using which approach to explain the patient's depression? A) Rogers's humanistic theory B) Beck's cognitive theory C) The opponent process model D) The diathesis-stress model

D - The answer to this question is D because the diathesis-stress model integrates the influence of biological predispositions and the environment.

If this study also included a functional neuroimaging procedure, then (compared to controls) patients with PTSD would be most likely to exhibit alterations in which of the following? I Prefrontal cortex II Hippocampus III Amygdala A) I and II only B) I and III only C) II and III only D) I, II, and III

D- The hippocampus, amygdala, and prefrontal cortex (anterior portion of the frontal lobe) are sensitive to stress hormones and demonstrate structural changes in response to chronic stress, such as what might be experienced with PTSD. (Number I) Prefrontal cortex: Chronic stress has been shown to decrease dendritic branching in areas associated with cognitive flexibility, resulting in reduced coping mechanisms. It also increases dendritic branching in areas associated with hypervigilance. (Number II) Hippocampus: Chronic stress has been shown to increase inflammation while decreasing dendritic branching and volume in the hippocampus. These physiological changes may inhibit learning and memory and produce emotional changes, such as anhedonia (decreased ability to experience pleasure). (Number III) Amygdala: Chronic stress changes the growth of dendritic spines and alters the excitability of neurons in the amygdala, leading to more anger, fear, and anxiety.

Mirror Neurons (passage Q)

Mirror neurons are a type of neuron that fires when an individual both observes and performs a behavior; they may play a central role in observational learning and the experience of vicarious emotions. Mirror neurons appear to be involved in grasping the intentions and in understanding the emotions of others. Deficiencies in the mirror neuron system are hypothesized to play a role in autism spectrum disorders. Vicarious emotions occur when an observer intuits the feelings that another might be experiencing. Vicarious emotions are thought to have evolved to help individuals relate to and understand one another through shared emotional expression. fMRI data suggest that neural activation when one experiences an emotion is similar to when one watches someone else experience that emotion, suggesting that vicarious emotions are linked to the mirror neuron system

Examples of monocular cues include the following:

Relative size & height of objects Interposition: in which one object blocks another Relative clarity: in which a more distant object appears blurry Relative motion: in which a more distant object moves more slowly Parallel lines converging Light & shadow

Examples of binocular cues include the following:

Retinal disparity: in which a greater difference between the two retinal images means the object is closer Convergence: in which the eyes facing at a more inward angle indicates the object is closer

Sensory receptors

Sensory receptors are specialized neurons capable of detecting environmental stimuli, including touch/pressure (mechanoreceptors), chemicals (chemoreceptors), heat (thermoreceptors), and light (photoreceptors).

Which statement is most consistent with a symbolic interactionist perspective? A) The definition for what is sacred may differ depending on the religion. B) Dynamic equilibrium is maintained when social control gradually shifts from religion to medicine. C) Individuals create meaning through serving others in faith-based mission work. D) Some religious groups attempt to reduce health care disparities that stem from class inequalities.

Symbolic interactionism is a micro-sociological perspective concerned with the interpersonal interactions through which the subjective meanings that shape social reality are developed. Subjective meanings differ by context and culture (eg, avoiding eye contact is a sign of respect in Japan but is considered rude in the US) and can change over time. The statement that individuals create meaning through serving (ie, interacting with) others in faith-based mission work is most consistent with symbolic interactionism. Under the symbolic interactionist perspective, mission work may become a meaningful expression and symbol of a believer's faith.

Unlike the descriptions of emotional processing in the second paragraph, the Cannon-Bard theory suggests that: A)Emotions do not require cognitive appraisal to be experienced. B) Emotional expressions are innate, not learned. C) Cognition must first occur for emotion to be felt. D) Physiological arousal and emotion occur simultaneously.

The Cannon-Bard theory of emotion suggests that physiological arousal and emotion are separate and independent and therefore occur simultaneously. For example, on seeing a wolf, an individual experiences physiological arousal (eg, increased heart rate) at the same time that the brain registers the emotion of fear.

Working Memory

Working memory and short-term memory are distinct executive functions. While both involve a limited and temporary store, working memory involves the manipulation and processing of information while short-term memory does not. Working memory is conceptualized as consisting of a central executive, which regulates attention and task switching, and three subsystems, which are controlled by the central executive. The visuospatial sketchpad is employed when manipulating visual and/or spatial information (eg, reading a map). The phonological loop is employed when manipulating spoken and written information (eg, reading a book). The episodic buffer is responsible for temporal processing (understanding the timeline of events) and integrating information from long-term memory into working memory (eg, remembering how to multiply when figuring out a tip at a restaurant). The results indicated that driving was impaired because of the cognitive overload produced by competing tasks and visual inattention. The central executive, responsible for attention and switching among various tasks, such as holding a conversation and driving, appears to be the aspect of working memory that is overloaded.

Cannon-Bard theory

states that our body's physiological response to a stimulus and our cognitive experience of emotion occur simultaneously but independently.

Female S. beldingi ground squirrels tend to remain near their birthplace and kin for life. S. beldingi females are also more likely than males to give alarm calls when they spot a predator, even though this behavior draws the predator's attention to the individual who produced the alarm. Which concept is LEAST relevant to this phenomenon? A) Autocommunication B) Altruism C) Indirect fitness D) Evolutionary game theory

A - Autocommunication occurs when a message sender is also the receiver. For example, dolphins echolocate by perceiving how the click sounds they have emitted echo back to them. Predator warning calls, communication that alerts other group members (not the individual making the call), is not relevant to autocommunication. (Choice B) Altruism is behavior that benefits others in a group at a potential cost or risk to oneself. For example, sounding an alarm call in response to a predator sighting benefits the group of S. beldingi squirrels, but places the individual who made the call at risk by drawing the predator's attention. (Choice C) An individual's inclusive fitness is the sum of its direct fitness (own reproduction) and indirect fitness(cooperative behavior that aids kin). Thus, inclusive fitness serves as a metric for an individual's total evolutionary success. Female S. beldingi squirrels, who remain near their birthplace and live by many relatives, engage in altruistic behaviors that maximize the chance that kin will survive (ie, indirect fitness). (Choice D) Game theory mathematically evaluates the relative success of particular strategies (behavioral phenotypes) over time. When the evolutionary benefit of a social behavior outweighs the cost of that behavior (eg, a warning call puts an individual at risk but increases the likelihood that genes will be passed on), game theory predicts that the behavioral phenotype will persist.

Which of the following symptoms, if present, would help a clinician diagnose bipolar disorder rather than borderline personality disorder? A) Lack of interest in activities previously enjoyed B) Impulsive or reckless behavior C) Fear of abandonment D) Suicidal ideation

A - Bipolar disorders (BD) are characterized by mania, defined as an abnormally elevated or irritable mood and increased energy. Manic episodes may also include feelings of grandiosity, reduced need for sleep, flight of ideas (rapid thoughts), and impulsive or reckless behaviors (eg, extreme spending, promiscuous sexual activity). Most people with BD also experience depressive episodes. Depressive episodes share the same symptoms as depressive disorder, such as feelings of sadness or hopelessness, lack of interest in activities, sleep disturbances, and frequent thoughts of death or suicide. Borderline personality disorder is characterized by instability in mood, sense of self, and relationships. Symptoms may include extreme mood reactivity, fear of abandonment, impulsive or reckless behavior (eg, drug use, unsafe sex), and suicidal or self-harming behavior.

Which of the following best exemplifies a primary circular reaction during the sensorimotor stage of development? A) An infant repeatedly vocalizing the same syllable B) An infant repeatedly banging their fist against the wall C) An infant understanding that a toy that is taken out of their view continue to exist D) An infant considering a person to disappear during a game when they hide their face

A - Circular reactions are named for their repetitive nature and primary are those involving a repetitive behavior which the child finds soothing and secondary are those which involve a repetitive behavior that involves and affects their environment. (B) is a secondary circular reaction while (A) is a primary circular reaction.

Stimulation of the nucleus accumbens would be expected to alleviate which of the following symptoms of depression? A) Anhedonia B) Difficulty concentrating C) Fatigue D) Insomnia

A - Depression is a mood disorder that is characterized by feelings of sadness, hopelessness, helplessness, low self-worth and self-esteem, fatigue, lack of interest in previously enjoyed activities, loss of motivation, changes in sleep patterns (sleeping more or sleeping less), changes in appetite (eating more or eating less), cognitive difficulties (concentration, decision making, memory), body aches, and lack of pleasure (anhedonia). Those suffering from major depression may also have suicidal ideation or commit suicide. The nucleus accumbens is a structure that is part of the reward pathway in the brain. The reward pathway also includes the ventral tegmental area (which produces dopamine) and portions of the prefrontal cortex. DBS of these structures could help alleviate symptoms of depression, as activation of the reward pathway is associated with feelings of pleasure and the reinforcement of rewarding behaviors.

Erica is an outgoing young woman who copes well with stress by maintaining order and responsibility in her life. Which combination of traits best describe Erica's personality? A) High extraversion, low neuroticism, high conscientiousness B) Low agreeableness, high neuroticism, low conscientiousness C) Low extraversion, low neuroticism, low openness D) High extraversion, high neuroticism, low conscientiousness

A - Erica is an outgoing (extroverted) young woman and demonstrates high extroversion (choice C is incorrect). Erica copes well with stress, thus demonstrating low neuroticism (choice B is incorrect). Finally, Erica is organized and orderly, demonstrating high conscientiousness (choice D is incorrect).

In an experiment, researchers find that certain neurons in the visual cortex preferentially fire in response to a bar of light that is oriented at a specific angle, and that different neurons similarly respond to bars of light oriented at different angles. This finding provides the strongest evidence for: A) Feature detection B) Parallel processing C) Spreading activation D) Sensory adaptation

A - Feature detection involves the perceptual discrimination of specific aspects of a given stimulus via feature detectors. Feature detectors are specific neurons that preferentially fire in response to very specific stimuli. Feature detection occurs for all the senses but is most often described regarding vision. Feature detectors in the visual system respond to aspects of the visual stimulus, such as horizontal lines or right angles (among others). The visual system is organized such that feature detectors synapse on neurons that respond to more complex stimuli (eg, faces) localized in certain areas of the brain (eg, fusiform face area). An experiment in which researchers find that certain neurons in the visual cortex preferentially fire in response to specific stimuli (eg, a bar of light oriented at a specific angle) provides the strongest evidence for feature detection. (Choice B) Parallel processing describes the brain's ability to simultaneously process various components (eg, color, motion) of a visual stimulus. This experiment did not demonstrate the simultaneous firing of different neurons detecting different components of a stimulus, so it does not provide strong evidence for parallel processing. (Choice C) Spreading activation occurs when a node (ie, concept) within an individual's semantic network (a uniquely organized cognitive web of information) triggers the activation of other, related nodes, a process known as priming. Because the experiment did not test priming, its findings do not provide evidence for spreading activation. (Choice D) Sensory adaptation is a diminished nervous system response over time to stimuli that remain constant, resulting in diminished stimulus perception (eg, a constant odor becomes less noticeable over time). This experiment did not present stimuli until the response diminished, so its findings do not provide evidence for sensory adaptation.

Which of the following is true, according to the Mead theory of identity development? A) The "me" is the aspect of self that is formed through interactions with others. B) The "I" is the aspect of self that is formed during the game stage. C) The "looking-glass self" results from an individual's cumulative lifetime of interactions with others. D) The "front-stage self" involves behaving in ways that align with social norms and expectations.

A - George Herbert Mead, who is most associated with the sociological theory of symbolic interactionism, argued that social (rather than biological) factors influence identity formation. Mead suggested that the experience of "self" emerges through social interaction with others who play important and formative roles in one's life (eg, family). The two aspects of the self are the "I" and "me," which develop in stages: preparatory, play, game.

Which of the following is true, according to the Mead theory of identity development? A) The "me" is the aspect of self that is formed through interactions with others B) The "I" is the aspect of self that is formed during the game stage C) The "looking-glass self" results from an individual's cumulative lifetime of interactions with others D) The "front-stage self" involves behaving in ways that align with social norms and expectations

A - George Herbert Mead, who is most associated with the sociological theory of symbolic interactionism, argued that social(rather than biological) factors influence identity formation. Mead suggested that the experience of "self" emerges through social interaction with others who play important and formative roles in one's life (eg, family). The two aspects of the self are the "I" and "me," which develop in stages: Preparatory (or imitation): Babies/toddlers imitate others (eg, a parent's hand gesture) and begin using symbols and language (eg, repeating a phrase used by a parent) without meaning comprehension. At this stage, children have no sense of "self" as separate from the world around them. Play: Through play (eg, pretending to be a doctor), preschool-age children begin role-taking (ie, understanding the perspectives of others). When children understand themselves as individuals separate from others, the "I" component of the self has developed. Children then begin to imagine how others perceive them, which is the beginning of the development of the "me." Game: School-age children become aware of their position/role in relation to others. They begin to see themselves from the perspective of the more abstract generalized other, further developing the "me" to incorporate the values and rules of the society in which they live. (Choice B) According to the Mead theory, the "me," not the "I," is formed through interactions with others during the game stage. (Choice C) Charles Cooley (not Mead) proposed the concept of the looking-glass self, suggesting that beliefs about oneself are derived through social interactions with others throughout one's lifetime. The Mead theory differs from the Cooley theory by suggesting that interactions early in life result in one's conception of self or identity. (Choice D) The dramaturgical perspective of Erving Goffman (not Mead) explains social behavior using a theater metaphor; the front-stage self involves individuals behaving as "actors" in front of an "audience," whereas the back-stage self includes the behaviors in which an individual engages more privately.

The increase of medicalization in society will result in medical professionals having: A) Greater power and authority. B) Reduced power and authority. C) Greater authority, but with no effect on their power. D) Greater power, but with no effect on their authority.

A - In sociology, power refers to the ability to influence and control others. Authority refers to whether others perceive that power as legitimate. As medicalization increases in a society, individuals will increasingly rely on medical professionals, rather than clergy, as legitimate sources of guidance on lifestyle behaviors. Therefore, medical professionals will have more power (ie, influence on people) and authority, as that power will be seen as legitimate.

A study found that over time, compared to white immigrants, immigrants of color experience poorer health overall. The researchers hypothesized that these immigrants, while living in the United States, are confronted with implicit and explicit negative messages about their outsider status as both a racial "minority" and as "immigrants," which have a cumulative impact on health. This hypothesis is most related to which pair of factors? A) Intersectionality and racialization B) Cultural transmission and racialization C) Intersectionality and stereotype threat D) Cultural transmission and stereotype threat

A - Intersectionality describes how individuals hold multiple, interconnected, marginalized social identities (eg, gender, race, age) that impact their lives, perspectives, and treatment in society. In other words, the experience of more than one type of discrimination (eg, sexism, racism, ageism) has a cumulative effect on many individuals. Racialization is the process by which one group designates another group with a racial identity, often based on shared group qualities, such as physical attributes (eg, skin pigmentation) or behaviors (eg, religious practices). The designating group has more social power (dominant group) and exerts social control over the designated group, which has less social power (subordinate group). The hypothesis that implicit and explicit negative messages about outsider status as both a racial "minority" and an "immigrant" have a cumulative impact on health is related to intersectionality (cumulative effects of racial "minority" and "immigrant" statuses) and racialization (implicit and explicit messages about one's racial "minority" status). (Choices B and D) Cultural transmission, which describes the passing of cultural information (eg, behaviors, beliefs, objects) from one generation to the next, is not related to the researchers' hypothesis. (Choices C and D) Stereotype threat occurs when task performance suffers as a result of being made aware before the task of a negative stereotype about a group to which one belongs (eg, awareness of the stereotype "women are bad at math" before a math task hinders women's performance). The researchers' hypothesis is unrelated to negatively impacted task performance.

Which of the following drugs has an effect on the central nervous system similar to that of alcohol? A) Benzodiazepines B) Hallucinogens C) Cocaine D) Amphetamines

A - Sedatives (eg, barbiturates, benzodiazepines) and alcohol belong to a category of drugs called depressants (or sedative hypnotics), which slow down, inhibit, or depress the central nervous system. These drugs are also known as anxiolytics as they reduce anxiety. Depressants exert their primary anxiolytic effect through binding at the GABAA receptor, enhancing the effect of GABA, the principal inhibitory neurotransmitter in the nervous system. (Choice B) Hallucinogens (eg, LSD) are not depressants; rather, they induce powerful mind-altering effects, including false sensory perceptions (hallucinations) and an altered sense of time. (Choices C and D) Cocaine and amphetamines are stimulants, which increase feelings of well-being, energy, and alertness.

Spatial disorientation experienced by pilots and divers would be LEAST affected by information processed by the: A) Organ of Corti. B) Otolith organs. C) Semicircular canals. D) Somatosensory system.

A - Spatial orientation and balance rely on input from the visual system, vestibular system (semicircular canals and otolith organs), and somatosensory system (proprioception - location and movement of our bodies in space). Sensory input is processed by the brainstem and cerebral cortex, and by the cerebellum, which coordinates balance. The semicircular canals detect angular acceleration, whereas the otoliths detect linear acceleration; The organ of Corti is located within the cochlea of the inner ear. Unlike the vestibular organs, the cochlea is not directly involved in processing spatial information. Instead, the main role of the cochlea is sound processing and transmission of auditory information to the temporal lobe of the brain.

Which of the following terms most accurately describes this patient's loss of friends and job as a result of his symptoms? A) Downward drift hypothesis B) Vicious cycle hypothesis C) Negative symptoms D) Delusions of reference

A - The downward drift hypothesis states that schizophrenia causes a decline in socioeconomic status and social skills, leading to worsening symptoms that cause the decline again. The vicious cycle hypothesis is not a psychological term (choice B is incorrect). Negative symptoms refer to the absence of normal behavior (choice C is incorrect). Delusions of reference are beliefs of the patient that environmental elements are targeting themselves in particular (choice D is incorrect).

Durkheim is most often associated with which sociological theory? A) Structural functionalism B) Conflict theory C) Social constructionism D) Symbolic interactionism

A - The major theoretical approaches in sociology view society from either a macro-level or micro-level perspective. Theories with a macro-level perspective look at how larger social structures (eg, education and health care systems) function in society and impact the lives of those living in that society. Theories with a micro-level perspective look at how interactions between individuals (eg, teacher and student, doctor and patient) shape society. Structural functionalism is a macro-level sociological perspective that compares society to a biological organism. This theory proposes that, like the various organ systems attempting to maintain homeostasis in an organism, all aspects of society work together to maintain dynamic equilibrium (societal balance). For example, the heath care system functions to keep people optimally healthy so that they can work and contribute to society, and the education system functions to teach and train people who will in turn contribute to society. Émile Durkheim is the theorist most often associated with structural functionalism. (Choice B) Karl Marx and Max Weber are most often associated with conflict theory, a macro-level sociological perspective which argues that conflict between social classes in society is inevitable. (Choice C) Social constructionism is either a micro- or macro-level perspective that argues that members of a given society define what constitutes reality. For instance, race is a social construct based arbitrarily on skin color. (Choice D) Charles Cooley and George Herbert Mead are most often associated with symbolic interactionism, which is a micro-level perspective that argues we attach meaning to symbols in society, and that interpersonal interaction is based on those symbols and meanings.

An older individual learns about the potential side effects of a drug she is currently taking from a pharmaceutical advertisement she sees on television. When she recalls the information later, she remembers hearing about those potential side effects from her physician, not the advertisement. This scenario illustrates which of the following? A) Source monitoring error B) Fundamental attribution error C) Negative priming D) Hindsight bias

A - The nature of memory construction leads to some common memory mistakes, such as source monitoring errors, which occur when a memory is attributed to the wrong source (eg, a cousin's funny story is mistakenly attributed to an aunt). The fundamental attribution error is the tendency to blame others' behavior on their internal (eg, "she is stupid") instead of external (eg, "the test was hard") factors. The fundamental attribution error does not account for misattributing information to the wrong source. Negative priming refers to the impaired processing (ie, slower response or reduced accuracy) that occurs when a stimulus is initially ignored and then later attended to (eg, first being asked to pick a red pen and ignore other colored pens in a cup, and then when asked to pick a blue pen, response time is slower). Negative priming does not account for misattributing information to the wrong source. Hindsight bias describes the tendency to perceive an event as being likely after it has occurred, even if it was unlikely (eg, after getting into a minor car accident, an individual states that she "just knew" she should have stayed home). Hindsight bias does not account for misattributing information to the wrong source.

A woman who wants a baby but is opposed to using medical intervention to become pregnant is informed by her doctor that she will never conceive without medical intervention. Cognitive dissonance theory predicts that this woman is most likely to do all of the following, EXCEPT: A) seek a second opinion from another doctor. B) change her attitude about using medical intervention to become pregnant. C) accept her doctor's diagnosis while maintaining her own opposition to medical intervention. D) decide that she does not want to have a baby.

According to cognitive dissonance theory, cognitive dissonance (mental conflict) results from beliefs, attitudes, or behaviors that are contradictory or incompatible. Cognitive dissonance causes a state of discomfort that results in motivation to reduce the conflict by aligning thoughts and/or behaviors. A woman who wants a baby but does not believe in medical intervention to conceive should experience cognitive dissonance if her doctor says she cannot become pregnant without such intervention, because the new information directly contradicts her belief. Therefore, cognitive dissonance theory predicts that she will alter her attitudes, beliefs, or behaviors to reduce the resulting discomfort. The only course of action that will not reduce the discomfort would be accepting her doctor's diagnosis and maintaining her stance against medical intervention. This course of action would maintain the dissonance, which is not consistent with the central tenet of cognitive dissonance theory.

The DSM-5 describes AN as characterized by a distorted body image and excessive food restriction leading to severe weight loss. How would a behaviorist explain the behavior of an individual diagnosed with AN? A) The eating behavior is evidence of a biochemical imbalance in the dopamine reward system in the brain B) Positive attention from others for initially losing weight motivates the individual to want to lose more weight C) The individual is behaving in a way to attain the "thin ideal" portrayed by celebrities in the media D) The individual most likely has a close friend or family member with an eating disorder and is imitating that behavior

B - Behaviorism is a theory in psychology that focuses on the role the environment plays in shaping human behavior. Specifically, behaviorism focuses on how reinforcement and punishment shape human behavior. Reinforcement is anything that increases the likelihood of the behavior happening again. For example, positive attention for losing weight would reinforce that behavior and make it more likely the individual will lose more weight. Punishment is anything that decreases the likelihood of a behavior happening again. For example, if individuals are ridiculed for a behavior, they are less likely to engage in that behavior again. Although positive attention for losing weight might initially reinforce the behaviors (eg, food restriction) associated with that weight loss, additional weight loss can become a self-reinforcing cycle in which an individual no longer needs external reinforcement and the weight loss itself is reinforcing. (Choice A) A behaviorist approach is more likely to be focused on the environmental factors influencing behavior whereas a biological approach is more likely to consider biochemical factors. (Choice C) A behaviorist might consider the role of idealized body types portrayed in the media and societal standards of beauty but would be more concerned with the direct reinforcements and punishments that shape individual behavior. (Choice D) A social learning theorist is more likely to suggest that individuals with AN have close friends or family members whose behavior they are imitating. Social learning theory suggests that behavior is shaped through social interaction, imitation, and modeling, and can occur in the absence of direct reinforcement.

Research has shown that when individuals consume the same food stimulus, they reach satiety faster compared to when they consume varied food stimuli. This finding suggests that satiety, which involves a reduction in both physiological and behavioral responses to food stimuli, may be partly due to: A) Desensitization B) Habituation C) Operant conditioning D) Generalization

B - Habituation, a type of nonassociative learning, occurs when repeated exposure to a stimulus results in a decreased response. For example, an individual who has had an intravenous line (stimulus) inserted into the arm might initially notice the feeling of it but will notice it less over time (diminished response). Appetite and satiety are influenced by internal and external cues. The research finding that when individuals consume the same food stimulus (ie, repeated exposure to a stimulus) they reach satiety (ie, diminished physiological and behavioral response) faster than when they consume varied food stimuli suggests that satiety may be partly due to habituation. (Choice A) Sensitization, another type of nonassociative learning, occurs when a stimulus produces an increasing response, and desensitization occurs when a response to a sensitized stimulus is reduced. The research findings did not show an increased response followed by a decreased response, so desensitization is not involved. (Choice C) Operant conditioning, a type of associative learning, occurs when the likelihood of repeating a behavior is influenced by reinforcement (increases likelihood) or punishment (decreases likelihood). The research findings did not show reinforcement or punishment influencing the future likelihood of a behavior, so operant conditioning is not involved. (Choice D) Generalization occurs in classical conditioning (a type of associative learning) when a conditioned response is elicited by stimuli similar to the original conditioned stimulus (eg, a baby conditioned to fear a white stuffed toy also fears white hair). The research findings did not show a conditioned response elicited by stimuli similar to the original conditioned stimulus, so generalization is not involved.

Which of the following brain regions is likely the most effective DBS target for a Parkinson disease patient? A) Amygdala B) Basal ganglia C) Hippocampus D) Motor cortex

B - Parkinson disease is a progressive neurodegenerative disease caused by death of dopaminergic neurons in the substantia nigra, located in the midbrain (mesencephalon). The substantia nigra is part of a group of neural structures collectively known as the basal ganglia, which are involved in the selection and execution of voluntary motor programs. Dopamine has several functions in the central nervous system, including mediating the reward pathway and motor control. The basal ganglia are composed of three dopaminergic pathways: the direct pathway, the indirect pathway, and the nigrostriatal pathway. The direct and indirect pathways have excitatory and inhibitory effects on motor function, respectively, and the nigrostriatal pathway helps maintain the balance between these two pathways. The nigrostriatal pathway is damaged when dopaminergic neurons in the substantia nigra are destroyed. Therefore, the balance between excitatory and inhibitory motor commands deteriorates. Resulting Parkinson symptoms include various motor abnormalities: resting tremor (usually in the upper extremities), muscle rigidity, slowed movement (bradykinesia), shuffling gait, and lack of coordination. DBS in portions of the basal ganglia has been shown to relieve some of these motor symptoms.

Which of the following brain regions is likely the most effective DBS (Deep brain stimulation) target for a Parkinson disease patient? A) Amygdala B) Basal ganglia C) Hippocampus D) Motor cortex

B - Parkinson disease is a progressive neurodegenerative disease that involves the loss of dopaminergic neurons in the substantia nigra of the basal ganglia, resulting in motor abnormalities (resting tremor, muscle rigidity, bradykinesia, shuffling gait, lack of coordination) and symptoms of depression.

Place theory is most appropriate for explaining how infants: A) Associated the red light with the auditory stimuli in Study 2. B) Processed the pitch of the auditory stimuli in Study 2. C) Distinguished the features of a human face in Study 1. D) Perceived the distance of a visual stimulus in Study 1.

B - Place theory explains the perception of sound pitch (how high or low a tone is). Inside the cochlea, specific sound wavelengths generate basilar membrane vibrations at specific loci. Each locus (place) corresponds to a slightly different frequency. Hair cells located at the base of the basilar membrane are activated by high frequency sounds, and hair cells located at the apex of the basilar membrane are activated by low frequency sounds. Place theory is most appropriate for explaining how the infants in Study 2 processed the pitch of the auditory stimuli.

Perception advertising is a kind of animal communication in which prey advertises that it has detected the predator, alerting the predatory to the fact that the surprise element has been lost. Which predator behavior is most directly affected by such communication? A) Mating behavior B) Foraging behavior C) Observational learning D) Near miss

B - Predators know that once the surprise element is lost, a chase is rarely successful. hence if the prey is able to communicate to the predatory that they have detected them, the predator is unlikely to embark on a chase, part of optimal foraging behavior. (D) near miss is a reference to the fact that Weber's law does not hold true for sound, and is not directly affected by such communication.

A researcher wants to determine whether proactive or retroactive interference is more affected by a specific type of traumatic brain injury. Which of the following research designs would best achieve their goals? A) A longitudinal study of six patients who are testing at regular intervals on their recall of two given patterns over a period of six months B) An experimental study of six patients who are asked to memorize Sequence A then Sequence B before having their recall tested C) A case study of one individual and the effect of the traumatic brain injury on forgetfulness in their daily life D) An observational study in which participants are paired together to freely interact while researchers document their ability to remember partners shared personal details

B - Proactive interference is the phenomenon in which memories of old information affects one's ability to remember new information. Retroactive interference is the phenomenon in which memorizing new information causes forgetting old information. Since researchers are looking to determine which is more affected, they need to do a study where both can occur and they can tell which process is more prevalent. (B) establishes an order of presentation, allowing researchers to determine whether they have an easier time recalling the original info (proactive) or second set (retroactive)

Which of the following statements about race is most consistent with the social constructionist perspective? In society, race is: A) Determined by genes B) Defined and maintained through social interactions. C) An achieved status. D) A social category used to maintain dynamic equilibrium

B - Social constructionism is a sociological theory suggesting that "reality" is created through social interactions, resulting in agreed-upon, shared meanings. Objects (eg, money), behaviors (eg, handshaking), and categories (eg, race) have meaning only because individuals in society have agreed on that meaning, making them social constructs. You chose D initially: From the functionalist perspective, which asserts that various parts of society work together to maintain dynamic equilibrium (ie, societal balance), race might be considered a social category that helps to maintain dynamic equilibrium, but this is not consistent with the social constructionist perspective.

Which of the following scenarios best represents intragenerational vertical mobility? A) An hourly wage employee has a son who becomes a neurosurgeon. B) A cardiologist loses her medical license and struggles to find a new job. C) A general surgeon in Denver moves his surgical practice to Dallas. D) A dentist inherits his mother's dental practice

B - Social mobility refers to the change or movement of individuals, groups, or families between or within status categories in society (eg, from middle class to upper class). Social mobility can be horizontal (ie, same social status) or vertical (ie, up or down in social status) and is related to a multitude of other factors, such as educational achievement, job loss, career advancement, marriage, and institutionalized discrimination. Intragenerational social mobility occurs within a single generation (ie, within a lifetime). Intergenerational social mobility occurs over multiple generations (ie, across several lifetimes). A cardiologist who loses her medical license and struggles to find a new job is an example of both intragenerational mobility, as this shift occurs within her own lifetime, and vertical (downward) mobility, because her social status decreases.

Disparities in prison demographics may be explained by all of the following, EXCEPT: A) Prejudice B) Stereotype boost C) Discrimination D) Confirmation bias

B - Stereotype boost (also known as stereotype lift) occurs when positive stereotypes about social groups cause improved performance. For example, Asians reminded of the stereotype "Asians are good at math" before completing a math test tend to outperform Asians who are not first reminded of this stereotype. This passage does not discuss improved performance as a result of a positive stereotype. Prejudice refers to preconceived ideas and beliefs about people or groups based on their group membership. Discrimination is the unequal treatment of individuals or groups on the basis of group membership. Confirmation bias is a type of cognitive bias (common error in thinking) in which individuals tend to embrace evidence supporting their beliefs, dismiss or ignore evidence refuting their beliefs, and interpret ambiguous evidence as support. Prejudice may cause an individual to act in a discriminatory fashion, creating the conditions for confirmation bias. (Choices A, C, and D) A police officer who believes black people commit more crimes (prejudice) and who spends the majority of his time patrolling predominantly black neighborhoods (discrimination) is more likely to witness black people than white people committing crimes, which confirms his original belief (confirmation bias).

Researchers did not recruit individuals in the age range that corresponds to the generativity vs. stagnation conflict in Erikson's theory of psychosocial development. Which statement provides the most likely explanation for the researchers' decision? A) The elderly have an increased risk of comorbid diagnoses, such as dementia, which complicate psychiatric diagnoses. B) Individuals in this age range would be older than the typical age of first active symptom onset in schizophrenia. C) Language and cognition typically merge in this age range, which could present a confounding variable. D) Interindividual variability in the progression of schizophrenia would limit the generalizability of the findings.

B - The age range corresponding to Erikson's generativity vs. stagnation stage is middle adulthood, or approximately 40-65. This stage is marked by feelings of productivity for individuals who have made successful contributions to society and feelings of stagnation for those who have not. The passage states that schizophrenia typically begins in early adulthood. Because the study aimed to test the ability of two different models to predict the future development of psychosis, a major component of schizophrenia, researchers did not recruit individuals in the age range corresponding to generativity vs. stagnation, as they would be older than the typical age of first active psychotic symptom onset in schizophrenia.

Suppose Jane is memorizing items on a list by associating them with stops on her path to the grocery store. Which memory tool is she using? A) Chunking B) Method of loci C) Peg-word D) Elaborative rehearsal

B - The method of loci involves the use of locations as a memory tool (choice B is correct). Chunking is a memory technique in which similar concepts are grouped together in smaller lists (choice A is incorrect). The peg-word technique refers to when items are associated with an alternate image or visual cue (choice C is incorrect). Elaborative rehearsal involves actively thinking about the meaning of the item trying to be remembered and linking it to other items in long-term memory (choice D is incorrect).

If a neuroimaging study compared normal dreaming with lucid dreaming, in what brain region would more activity be expected during lucid dreaming? A) Cerebellum B) Prefrontal cortex C) Amygdala D) Primary visual cortex

B - The prefrontal cortex is associated with executive functions, such as critical thinking, problem solving, planning, impulse control, and executive decision-making. Because lucid dreaming involves critical analysis and executive decision making, a neuroimaging study would be expected to demonstrate greater activation of the prefrontal cortex during lucid dreaming as compared with normal dreaming. The prefrontal cortex is the anterior portion of the frontal lobe, which also contains the primary motor cortex (responsible for the initiation of voluntary movement) and Broca area (involved in speech production). (Choice A) The cerebellum is positioned underneath the occipital lobe at the back of the skull and is associated with motor coordination and motor learning. This region would not necessarily be more active during lucid dreaming compared with normal dreaming. (Choice C) The amygdala is part of the limbic system located on either side of the thalamus deep in the cortex; it is associated with processing emotion, particularly fear and anger. Although this region may be active during lucid dreaming, it is probably no more active than during normal dreaming. (Choice D) Located in the occipital lobe, the primary visual cortex is associated with processing visual input from the retinas and is not expected to be as active during dreaming, when there is no external visual input. However, secondary visual areas of the brain have been associated with the visual imagery that occurs during dreaming.

According to the early theory of emotion described in the second paragraph, which of the following brain structures is most critical to the experience of emotion? A) Prefrontal cortex B) Hypothalamus C) Hippocampus D) Posterior pituitary

B - egions involved in emotion, learning, and memory. The limbic system includes the amygdala, hippocampus, hypothalamus, and cingulate gyrus. The hypothalamus regulates the pituitary gland and the autonomic nervous system. It controls the release of hormones that maintain homeostatic functions (parasympathetic "rest-and-digest" response) or prepare the body to respond to a stressor (sympathetic "fight-or-flight" response). The hypothalamus is responsible primarily for the physiological component of emotion, such as changes in heart or respiration rate. The James-Lange theory of emotion (second paragraph) suggests that specific physiological reactions produce specific emotions. For example, if experiencing increased heart rate and sweating, one will experience fear. Therefore, the hypothalamus is most critical to the experience of emotion according to the James-Lange theory. (Choice A) The prefrontal cortex plays a role in what is thought to be a uniquely human capability: the modulation of emotions. In other words, the prefrontal cortex helps regulate emotional responses, such as dampening anger so that, for example, lashing out at bosses does not occur. The modulation of emotion involves cognitive appraisal of the situation, which is not an aspect of the James-Lange theory. (Choice C) The hippocampus is involved primarily in memory consolidation, which is not a critical component of the James-Lange theory. (Choice D) The posterior pituitary, a collection of hypothalamic axonal projections, releases the hormones oxytocin and vasopressin. Oxytocin is involved primarily in attachment and pair bonding. Vasopressin, or antidiuretic hormone, regulates the reabsorption of water from the kidneys. Neither hormone appears to be as critical to the experience of emotion as those involved in the fight-or-flight response.

The following graph, depicting the foreign-born (immigrant) population in the United States (US) from 1850 to 2010, best supports which of the following conclusions? The graph shows that the # of immigrants have increased while the % has decreased* A) Compared to 1910, there was a decrease in pull factors driving immigration to the US in 2010 B) From 1910 to 1930, the fertility rate of the US population exceeded that of the immigration rate C) From 1940 to 1970, the mortality rate of the US foreign-born population declined D) From 1980 to 2010, the foreign-born fertility rate exceeded the native-born fertility rate

B -This graph displays the number of immigrants and the percentage of the total US population composed of immigrants from 1850 to 2010. From 1910 to 1930, the number of immigrants (graph bars) increased, whereas the percentage of immigrants in the overall US population (graph line) decreased, which would occur only if the native-born population increased faster than the foreign-born (immigrant) population. A fertility rate exceeding the immigration rate would produce this outcome.

Researchers conduct an experiment using dogs. Each dog is placed in a cage with a short middle partition. A light turns on five seconds before the dog receives a painful electric shock through the floor, which stops when the dog jumps over the partition. If researchers count the number of trials each dog needs until it jumps over the partition before the shock is administered, they are most likely studying: A) Escape learning. B) Avoidance learning. C) Habituation. D) Sensitization.

B- Escape learning occurs when an organism learns how to terminate an ongoing unpleasant stimulus (eg, a dog jumps over a partition to flee from or stop a continuous electric shock). Escape learning becomes avoidance learning when an organism prevents coming into contact with an unpleasant stimulus (eg, a dog jumps over a partition to avoid the electric shock before it occurs). Because the researchers are counting the number of trials each dog needs until it jumps over the partition before the shock is administered (ie, avoid the shock), they are studying avoidance learning.

Which of the functions is LEAST regulated by a structure in the brain? A) Involuntary breathing B) Reflexive motor responses C) Vestibular sense D) Blood pressure

B- Motor reflexes are caused by interactions between the peripheral nervous system and the spinal cord, not the brain.

Psychoanalytic theory would most likely suggest that experimental group participants who are able to alter their personality traits have: A) Superegos that overcome defense mechanisms. B) Ids that contain an intact reality principle. C) Stronger egos than participants who are not able. D) More rational ids than participants who are not able.

C - According to psychoanalytic theory (most associated with Sigmund Freud), personality results from the interaction between the id, ego, and superego. The id is selfish, compelling us to seek pleasure and avoid pain. The ego is realistic, compelling us to behave in ways that are socially acceptable, and acts as a mediator between the id and superego. The superego is moralistic and idealistic, compelling us toward perfection. According to psychoanalytic theory, personality change is possible when a person has a strong ego that can successfully mediate between one's impulsive, pleasure-seeking id and the goals of the idealistic superego. (Choice A) Defense mechanisms are unconscious strategies used by the ego when it is overwhelmed or otherwise weaker than the id. Defense mechanisms help alleviate anxiety or other suffering. The superego does not engage in defense mechanisms.

Which of the following cognitive biases is most analogous to the self-positivity bias? A) Actor-observer bias B) Confirmation bias C) Optimism bias D) Overconfidence bias

C - Cognitive biases are irrational thought processes that commonly occur and result in illogical conclusions. The self-positivity bias occurs when people believe that they are less vulnerable to negative outcomes than other people. The optimism bias is very similar, and describes the tendency for people to underestimate the probability that bad things (eg, cancer, car accident) will happen to them. The two cognitive biases are most similar because each describes the human predilection for miscalculating the likelihood that adverse events will impact them. (Choice A) The actor-observer bias occurs when individuals attribute other people's behavior to internal causes (eg, she yelled at her child because she's a terrible mother) while attributing their own behavior to external factors (eg, I yelled at my child because I had a bad day at work). (Choice B) The confirmation bias is the tendency to look for information that supports the conclusion one has already reached, to ignore information undermining that conclusion, and to interpret ambiguous information as supporting the conclusion. (Choice D) The overconfidence bias occurs when the degree to which people are sure of their belief is greater than the accuracy of that belief (ie, people overestimate their subjective knowledge compared to objective facts).

Which of the following is true regarding two classes of drugs commonly used to treat depression, selective serotonin reuptake inhibitors (SSRIs) and monoamine oxidase inhibitors (MAOIs)? A) Both classes of drugs stimulate the release of serotonin into the synaptic cleft B) SSRIs block uptake of serotonin into the post-synaptic neuron C) MAOIs decrease the breakdown of serotonin within the pre-synaptic neuron D) MAOIs promote uptake of serotonin into the pre-synaptic neuron

C - Depression symptoms appear to be related to decreased levels of the monoamine neurotransmitters serotonin, dopamine, and norepinephrine in the central nervous system (monoamine hypothesis). Many antidepressant medications increase the concentration or prolong the function of one or more of these neurotransmitters at the synaptic cleft. These drugs usually function via one of four general mechanisms: 1. Increasing production of neurotransmitters within the presynaptic neuron 2. Promoting release of neurotransmitters from the axon terminal into the synaptic cleft 3. Blocking reabsorption (reuptake) of neurotransmitters into the presynaptic neuron 4. Decreasing breakdown of neurotransmitters within the presynaptic neuron For instance, MAOIs inhibit monoamine oxidase, an enzyme attached to the mitochondria in axon terminals that catalyzes the oxidation (breakdown) of monoamines such as serotonin, dopamine, and norepinephrine. This enzyme functions in the presynaptic neuron to recycle monoamines that have been removed from the synaptic cleft. Inhibition of monoamine oxidase therefore increases the concentrations of monoamines in the axon terminal and eventually within the synaptic cleft. SSRIs belong to another class of antidepressants that selectively block the reabsorption (reuptake) of serotonin into the presynaptic neuron, thereby prolonging the presence of serotonin in the synaptic cleft. (Choice A) Neither SSRIs nor MAOIs directly stimulate the release of serotonin or other monoamines. However, some drugs, such as serotonin-releasing agents and amphetamines, promote the release of neurotransmitters to increase the activity of these in the synaptic cleft. (Choice B) Reuptake inhibitors such as SSRIs block the uptake of neurotransmitters into the pre-synaptic cleft, where they were originally synthesized and released. (Choice D) Promoting uptake of serotonin would decrease the concentration of serotonin in the synaptic cleft, likely exacerbating the symptoms of depression.

Bipolar I and bipolar II disorders are two distinct psychological disorders, characterized by which of the following differences? A) Bipolar I must include manic episodes and at least one major depressive episode B) Bipolar I must include a combination of hypomania and dysthymia C) Bipolar II must include hypomania and at least one major depressive episode D) Bipolar II must include manic episodes and can or cannot include one major depressive episode

C - Direct therapy is focused on the individual who is suffering rather than their family and can come in forms such as medicine or individual therapy (choice C is correct). As the name states, the biopsychosocial approach accounts for all three factors in diagnosing and treating psychological disorders (choice B is incorrect). The biomedical approach is completely rooted in the individual's biology and only focuses on somatic factors as the cause (choice A is incorrect). Indirect therapy is focused on providing a support system to the suffering individual and informs the family and friends of ways to help (choice D is incorrect).

Which of the following symptoms is LEAST characteristic of a dissociative disorder? A) Recurrent gaps in memory and more than one distinct personality B) Inability to remember details of a traumatic experience C) Fabricated memory loss in order to gain sympathy from others D) Difficulty remembering important autobiographical information

C - Dissociative disorders are characterized by disruptions to memory and identity. Two specific types of dissociative disorders, dissociative amnesia and dissociative identity disorder (DID), both feature the inability to recall important autobiographical information. This amnesia may be generalized for identity-related information and history (eg, name, marital status) or localized for specific events (eg, a traumatic experience). Factitious disorder is a somatic symptom and related disorder (SSRD) that involves falsifying physical or psychological symptoms without obvious external gain (eg, disability benefits). Fabricated memory loss to gain sympathy from others is characteristic of factitious disorder (an SSRD); therefore, this symptom is least characteristic of a dissociative disorder. (Choice A) In addition to gaps in memory, DID is characterized by the presence of two or more distinct personalities. Individuals with DID experience a discontinuous sense of self. (Choices B and D) The amnesia characteristic of dissociative amnesia and DID may involve an inability to remember the details of a traumatic experience or difficulty remembering important autobiographical information.

A group of five health care professionals is presented with a complex system-based problem in a large hospital and asked to generate multiple possible solutions. Research on group processes suggests that: A) If group members get along, more options will be discussed and considered. B) The group is more likely to come up with an effective solution if there is a strong group leader. C) If group members start with similar opinions, they will support an option that is more extreme. D) On average, individuals working alone will produce fewer options than when working in a group.

C - Group polarization occurs when group members adopt a more extreme attitude or course of action after group discussion. Group polarization is more likely to occur if group members have similar opinions before group discussion. Group discussion strengthens and reconfirms individuals' opinions, resulting in an average viewpoint that is more extreme. Group polarization can occur in either direction (ie, extremely positive or extremely negative).

A proponent of social cognitive theory would suggest that the self-positivity bias is unlikely to influence an individual: A) With hepatitis A. B) Studying hepatitis C in a liver cell line. C) With several friends who have hepatitis C. D) Who has never heard of hepatitis C.

C - Social cognitive theory is most concerned with how individuals learn through observing the behaviors of others. Therefore, a proponent of this theory would be concerned with how an individual's behavior and attitudes are shaped by interacting with others, rather than through personal experience with a similar disease, professional experience with the disease, or lack of knowledge about the disease.

How does stereopsis contribute to the processing of two-dimensional retinal images into objects having three-dimensional depth? A) The presence of rods and cones allows for processing different types of light. B) Component attributes, such as color, motion, and form, are integrated in the occipital lobe. C) The relative location of each retina allows for different images of the object to be processed. D) The movement of ciliary muscles controlling the lens provides depth cues to the cortex.

C - Stereopsis is the perception of depth that arises from the integrated information received from both eyes. Binocular depth cues allow accurate perception of depth through the integration of slightly different information from the left and right eyes. Binocular depth cues include retinal disparity and convergence: Retinal disparity, or distance between the two eyes, results in slightly different images projected onto the respective retinas. The closer an object or scene is to the two eyes, the more dramatic the disparity. Higher-order processing in the brain compares the difference between the two-dimensional retinal images to help give rise to the perception of three-dimensional depth. Convergence occurs because of the angular positioning of the eyes: The closer an object is to the observer, the more the eyes turn inward (or converge) to focus on that object. The brain interprets the angle of convergence as an indication of distance from the observer. Binocular cues are less informative at great distances because retinal disparity and convergence decrease as distance increases. Therefore, monocular cues become more important for depth perception over great distances.

How would a proponent of Malthusian theory most likely interpret the data in Figure 1? A) Increased suicide rates provide evidence for a Malthusian catastrophe B) The decline in suicide rates during World War II supports Malthusian theory C) The Great Depression would be considered a positive check D) Increased suicide rates during recessions are negative checks

C - The Malthusian theory of population growth suggests that the human population increases exponentially while resources increase at a slower rate. According to this theory, the population growth rate can be slowed by preventative checks and positive checks. Preventative checks are those that decrease the birth rate (and are typically voluntary, such as waiting to marry and having fewer children). Positive checks increase the death rate, slowing population growth by shortening the average life span. Positive checks can be small-scale (eg, increased death rate due to a flu virus) or large-scale (eg, an epidemic that wipes out half the population). Large-scale positive checks, called Malthusian catastrophes (eg, widespread famine, disease epidemics, large-scale wars), dramatically reduce the population to a level that the available resources can easily sustain, by slowing or stopping population growth entirely. (Choice A) Increases in suicide rates, even up to the highest rates in Figure 1 (15-20 suicides per 100,000 people), will not have a large enough impact on the population to be considered a Malthusian catastrophe. (Choice B) According to Malthusian theory, the increased death rate (not the decreased suicide rate) during World War II would be considered a positive check, supporting the theory. (Choice D) A negative check does not exist in Malthusian theory, only positive checks and preventative checks.

According to the elaboration likelihood model, which of the following should generate the most concern about the risks of contracting hepatitis C in the two-uncommon-behaviors group from Study 2? A television commercial showing: A) Text detailing the scientific facts about contracting hepatitis C. B) Actual drug addicts sharing needles. C) An attractive celebrity using a catchy phrase to summarize the main hepatitis C risk factors. D) A medical expert from a prestigious university summarizing the main hepatitis C risk factors.

C - The elaboration likelihood model defines two routes of persuasion: The central route (focusing on the logical content of the message) is most effective when the audience is motivated by the message, whereas the peripheral route (focusing on superficial characteristics of the message) is more effective when people are not motivated by the message. Detailed scientific facts and information presented by experts rely on the central route of processing, which is most effective for receivers who are highly motivated by the message. The results of Study 2 suggest that the uncommon behaviors groups are not highly concerned about contracting hepatitis C. According to the elaboration likelihood model, the most persuasive strategy for people who have low motivation and/or ability to process the message (such as the uncommon behaviors groups in Study 2) is to use the peripheral route of processing. Therefore, a television commercial with an attractive celebrity using a catchy phrase to summarize the main hepatitis C risk factors would be the best way to structure a persuasive health message about hepatitis C to this group.

A survey of medical students found that most believe the surgical specialty has its own clearly defined "masculine" subculture, and that to succeed as surgeons, individuals need to "adopt masculine characteristics." These beliefs best reflect which aspect of surgical medical education? A) Teacher expectancy B) Manifest functions C) Hidden curriculum D) Material culture

C - The formal curriculum includes the explicit, official content taught to students (eg, how to solve algebra equations). The hidden curriculum includes the implied, informal mechanisms by which certain values and behaviors are promoted; for example, an algebra teacher who calls mainly on male students is reinforcing a stereotype that boys are better at math than girls. Survey results indicating that the surgical specialty has a "masculine" subculture, such that "masculine characteristics" are needed for success, best reflect the hidden curriculum of surgical medical education.

In which region of the brain would a lesion most likely disrupt the sleep/wake cycle? A) Anterior pituitary B) Posterior pituitary C) Hypothalamus D) Hippocampus

C - The hypothalamus is centrally located in the brain and is the command center for the endocrine system, which produces hormones that regulate a number of the body's functions, such as growth, metabolism, blood pressure, core body temperature, appetite, and sleep. The hypothalamus has several nuclei (collections of neuronal cell bodies) that have specialized functions; one of these nuclei is the suprachiasmatic nucleus (SCN), which regulates the circadian pacemaker that controls circadian rhythms. Photoreceptors in the retina project information about light levels to the SCN. When light levels are high, the SCN downregulates melatonin production by the pineal gland. When light levels are low, the SCN upregulates melatonin production by the pineal gland. Light levels regulate SCN activity, which regulates melatonin release and establishes an internal circadian clock. This clock mechanism helps maintain sleep patterns and other 24-hour circadian cycles, such as those involving blood pressure and core body temperature changes. (Choices A and B) The pituitary sits below and is attached to the hypothalamus via the pituitary stalk. The anterior pituitary, controlled primarily by the hypothalamus via the hypophyseal portal system, synthesizes and secretes many hormones (eg, follicle-stimulating hormone, growth hormone). The posterior pituitary, made of axonal projections from the hypothalamus, secretes oxytocin and vasopressin (antidiuretic hormone). (Choice D) The hippocampus is a brain structure located within the middle temporal lobe and is associated with the formation and storage of memory.

Which of the following cognitive biases is most analogous to the self-positivity bias (when people tend to believe that they are less vulnerable to negative outcomes or events than others)? A) Actor-observer bias B) Confirmation bias C) Optimism bias D) Overconfidence bias

C - The optimism bias is very similar, and describes the tendency for people to underestimate the probability that bad things (eg, cancer, car accident) will happen to them. The actor-observer bias occurs when individuals attribute other people's behavior to internal causes (eg, she yelled at her child because she's a terrible mother) while attributing their own behavior to external factors (eg, I yelled at my child because I had a bad day at work). The confirmation bias is the tendency to look for information that supports the conclusion one has already reached, to ignore information undermining that conclusion, and to interpret ambiguous information as supporting the conclusion. The overconfidence bias occurs when the degree to which people are sure of their belief is greater than the accuracy of that belief (ie, people overestimate their subjective knowledge compared to objective facts).

Cognition

Cognition refers to thought-related mental processes (eg, thinking, planning, reasoning) and is believed to be linked to language. There are several major theories that deal with the relationship between language and cognition: Cognition shapes language: These theories assert that cognition either completely controls language (universalism perspective) or just influences language (Piagetian perspective). Language shapes cognition (also known as the Sapir-Whorf hypothesis): These theories suggest that language either just influences cognition (linguistic relativity perspective) or completely controls cognition (linguistic determinism perspective). Language and thought develop independently and simultaneously: Vygotsky suggested that social interaction precedes and is necessary for both cognition and language development, which are initially separate but then later merge to generate inner speech (verbal thought). Universalism, the belief that cognition controls language, posits that certain cognitive processes are universal, so all human languages also possess certain universals (eg, nouns). Therefore, a universalist would interpret Figure 1B as support for the belief that cognition is required for language development.

A researcher is testing a patient's neurological function. When a word is flashed briefly in the patient's right visual field, he can correctly vocalize what he saw. When a word is flashed briefly in the patient's left visual field, he is unable to say what he saw but can correctly draw it. Which of the following is most likely damaged in this patient? A) Left retina B. Wernicke area C. Right occipital cortex D. Corpus callosum

D - Corpus Callosum Damage to the right occipital cortex would result in an inability to process visual information presented in the left visual field. This patient is still able to draw the image represented by the word even though he cannot speak it, suggesting that the occipital lobe is intact. The corpus callosum allows communication between the right and left hemispheres of the brain, which are specialized for certain functions (cortical lateralization). Each hemisphere contralaterally controls the opposite side of the body. Language centers are located in the left hemisphere.

Which of the following best illustrates a normative organization? A) A residency program for physicians specializing in surgery B) An inpatient psychiatric ward where patients are involuntarily committed C) An insurance company employing physicians as paid consultant D) A humanitarian group composed of volunteer physicians

D - In sociology, organizations are social structures composed of individuals linked by specific purposes, tasks, and/or common goals. The three main types of organizations are normative, utilitarian, and coercive. In normative organizations, people voluntarily unite based on shared values and/or goals (eg, church congregations, sororities). In utilitarian organizations, members are compensated for their involvement, typically through money (eg, paid employment) or certification/diploma (eg, university students). In coercive organizations, membership is not freely chosen (eg, prisoners) and/or maintained (eg, military service members must be discharged). A humanitarian group composed of physician volunteers illustrates an organization in which members are united by shared values and/or goals (ie, a normative organization).

Which receptor is NOT involved in proprioception? A) Muscle spindle B) Golgi tendon organ C) Vestibular apparatus D) Baroreceptor

D - Muscle spindles and Golgi tendon organs, located in skeletal muscles, are both involved in proprioception and relay information to the brain about the body's position in space (choices A and B are incorrect). The vestibular apparatus is a proprioceptor located in the inner ear that relays information about the position of the head by detecting the movement of cochlear fluid (choice C is incorrect). Baroreceptors are mechanical receptors that detect changes in pressure. Thus, they are NOT involved in proprioception

If the survey from Study 3 that assessed the patients' level of AN was determined to be reliable, which of the following statements is true? A) Researchers can confidently extrapolate from Study 3 to all patients with cancer. B) Survey administration is consistent across medical settings. C) The survey is an accurate measure of nausea symptoms. D) Nausea scores on the survey will be similar from one survey administration to the next.

D - Reliability describes the degree to which a measure or experiment is consistent (eg, a test produces a similar score for one person who takes it twice). Validity refers to the accuracy of a measure or experiment (eg, a test accurately assesses what it was designed to measure).

Which of the following is the LEAST concerned with the role of nonverbal communication in influencing the social behavior of individuals? A) Elaboration likelihood model B) Dramaturgical perspective C) Symbolic interactionism D) Structural functionalism

D - Structural functionalism is a macro-level (large-scale) sociological perspective suggesting that all aspects of society work together to maintain dynamic equilibrium (societal balance). Structural functionalism is not concerned with micro-level (small-scale) interactions between individuals, so this theory would be the least concerned with how nonverbal communication influences the social behavior of individuals. The elaboration likelihood model, dramaturgical perspective, and symbolic interactionism each deal with how individuals interact with and attempt to influence each other, partly through the use of nonverbal cues.

Performance of which of the following study tasks utilizes the mirror neuron system? Immediate recall task Delayed recall task Distractor task A) I only B) III only C) II and III only D) I, II, and III

D - Studies suggest that mirror neurons are specialized neurons in the brain that fire both while observing and while performing a behavior. Watching and replicating movements, such as during the Corsi block-tapping task, is a form of observational learning, which is thought to utilize the mirror neuron system. (Number I) As part of the Corsi block-tapping task, the immediate recall task requires subjects to touch the same blocks as the experimenter immediately afterward, which would rely on the mirror neuron system. (Number II) The delayed recall task requires subjects to touch the same blocks the experimenter touched after a delay of 5-25 seconds; this is an example of observational learning and would also rely on the mirror neuron system. (Number III) The distractor task requires subjects to watch and reproduce a finger-tapping sequence first demonstrated by the experimenter, which would also rely on the mirror neuron system.

The Cannon-Bard theory suggests that: A) Emotions do not require cognitive appraisal to be experienced B) Emotional expressions are innate, not learned C) Cognition must first occur for emotion to be felt D) Physiological arousal and emotion occur simultaneously

D - The Cannon-Bard theory of emotion suggests that physiological arousal and emotion are separate and independent and therefore occur simultaneously. For example, on seeing a wolf, an individual experiences physiological arousal (eg, increased heart rate) at the same time that the brain registers the emotion of fear. Cannon and Bard conducted experiments on cats in which they severed the sympathetic afferent neurons to the brain and then exposed the cats to fear-inducing stimuli. If emotion is caused by physiological arousal (James-Lange theory), then the cats should not demonstrate fear without autonomic feedback from their bodies. Since they did demonstrate fear, Cannon and Bard theorized that the two responses occurred simultaneously and independently. The Cannon-Bard theory of emotion would explain the events described in the second paragraph by arguing that physiological arousal and the experience of emotion all occur simultaneously. (Choice A) Neither the James-Lange nor the Cannon-Bard theory of emotion takes cognitive appraisal into account. The suggestion that emotion requires cognitive appraisal to be experienced is consistent with the Schachter-Singer theory of emotion, which suggests, like the James-Lange theory, that physiological arousal precedes emotion but also suggests that cognitive interpretation of the situation allows the correct labeling of the emotion by the brain. (Choice B) Neither the James-Lange nor the Cannon-Bard theory deals with the innateness of emotion. The idea that emotional expressions are innate and unlearned is consistent with the evolutionary theory of emotion. (Choice C) The idea that cognition must first occur for emotion to be felt is consistent with the cognitive appraisal theory of emotion, not the Cannon-Bard theory.

Researchers have identified a set of factors associated with an increased risk of developing chronic obstructive pulmonary disease, including low birth weight, respiratory infections before age 2, low socioeconomic status in childhood, adolescent smoking, and occupational exposure to airborne irritants as an adult. This finding best reflects: A) The sick role theory approach B) The illness experience perspective C) The social construction of chronic disease D) The life course approach to illness

D - The life course approach is a holistic, multidisciplinary framework for understanding how psychological, biological, and sociocultural factors across a lifetime have a cumulative effect on health outcomes. This perspective considers how personal life events (eg, illness in infancy), individual choices/behaviors (eg, having unprotected sex), and sociocultural and historical context (eg, being born during wartime) impact health and illness. Research suggesting that lifetime exposure to biological (eg, respiratory infections), sociocultural (eg, low socioeconomic status), and behavioral (eg, smoking) risk factors increases the likelihood of developing chronic obstructive pulmonary disease (COPD) is most reflective of the life course approach to illness. (Choice A) Sick role theory is a functionalist approach describing how disruption to typical social activity (eg, work) caused by illness is minimized through the sick role, which legitimizes illness as socially acceptable deviance. Biological and sociocultural factors impacting health/illness across a lifetime are not relevant to sick role theory. (Choice B) The illness experience is a symbolic interactionist approach to understanding how people incorporate and make sense of illness as part of their self-identity and daily routines. The illness experience is not best reflected by evidence that biological and sociocultural factors impact health/illness across a lifetime. (Choice C) Social constructionism suggests that social reality is created through interpersonal interactions, which result in shared meanings and expected roles/behaviors. The social construction of chronic illness would not be concerned with the cumulative impact of factors across a lifetime contributing to risk of chronic illness.

A cognitive-behavioral therapist in Study 2 uses principles of classical and operant conditioning to treat a participant with extreme introversion and social anxiety. The methodologies employed by the therapist would LEAST likely include: A) Repeatedly exposing the participant to extended periods of social engagement. B) Commending the participant for attempts to engage socially during free time. C) Requiring the participant to eat favorite foods only when in the presence of company. D) Asking the participant to observe others who freely engage in social settings

D - The primary objective of CBT is to replace negative thought patterns and destructive behaviors with healthy, positive thoughts and behaviors. A cognitive-behavioral therapist treating a participant with extreme introversion and social anxiety would focus on replacing negative thoughts (eg, "I'm no good in social situations") and avoidant behaviors (eg, not attending a party) with more positive thoughts (eg, "I'm interesting and a great listener") and sociable behaviors (eg, eating with others). If relying on principles of classical conditioning, a CBT therapist might use a form of systematic desensitization, in which this participant is methodically exposed to longer periods of social engagement (Choice A). A CBT technique that relies on principles from operant conditioning is rewarding social behaviors (Choices B and C). Asking the participant to observe others who freely engage in social settings, while at times used in CBT, is based more on the principles of observational learning than on those of classical or operant conditioning.

Which of the following best summarizes a difference between Korsakoff Syndrome (KS) and Alzheimer Disease (AD)? A) KS has a physiological cause, but AD does not. B) KS is caused by environmental insult, but AD is genetic. C) KS is associated with biochemical changes in the brain, but AD is not. D) KS can be reversible, but AD is irreversible

D - The umbrella term dementia describes a decline in one or more aspects of cognitive functioning (eg, memory, attention, executive functioning, perception, motor organization, language, social cognition). Declines result in poorer social, occupational, and/or daily functioning and can be associated with personality changes and difficulty regulating emotions. Although dementia can be caused by a range of diseases or brain injury, dementia due to Alzheimer disease (AD) is the most common type. Dementia due to Korsakoff syndrome (KS) is caused by a deficiency in thiamin (vitamin B1), typically resulting from chronic alcohol consumption (alcohol use disorder). People with alcohol use disorder generally have poor diets, and alcohol interferes with the absorption of thiamin in the digestive tract. Thiamin supplementation and behavior modification in the early stages of KS can potentially reverse most brain damage, but AD is irreversible, ultimately progressing to global cell death and severe brain atrophy.

The neuroimaging tool used for this study (fMRI) works by measuring which of the following? A) Electrical impulses B) Glucose metabolism C) Radioactive tracing D) Blood oxygenation

D - This study used functional magnetic resonance imaging (fMRI), a neuroimaging procedure that relies on the principle that active neurons require increased blood flow for oxygen delivery. Neurons lack internal energy stores (ie, no glucose or oxygen), so as an active brain region depletes the local oxygen supply in the blood, freshly oxygenated blood (oxyhemoglobin) rushes to this area to replace the deoxygenated blood (deoxyhemoglobin). The resulting change in blood flow is analyzed by detecting the differential properties of oxyhemoglobin and deoxyhemoglobin using an fMRI scanner. Researchers use this method to compare blood flow during a resting condition and an active condition. For example, the study researchers would have first assessed subjects at baseline (resting condition) and then while watching videos meant to induce emotion (active condition) and compared the regions of brain activation. (Choice A) An electroencephalogram (EEG) measures the patterns and fluctuations of electrical impulses in the brain through electrodes placed on the scalp. This technique records brain waves, and is most often used to diagnose epilepsy, sleep disorders, and coma. (Choices B and C) Positron emission tomography (PET) uses a radioactive tracer attached to a glucose analog to measure blood flow and glucose uptake induced by brain activity. PET is often combined with other procedures and produces a three-dimensional image of the brain showing hypermetabolic and hypometabolic regions.

A therapist treats a patient with agoraphobia. if the therapist subscribes strongly to the tenets of cognitive-behavioral therapy, which of the following treatments would be most representative of the therapist's view of the causes of the disorder? A) Showing the patient pictures of crowded situations while playing soothing music B) Preparing the patient with relaxation techniques before having her spend a day in a busy airport C) Prescribing a serotonin reuptake inhibitor to treat the symptoms of anxiety generally D) Accompanying the patient to the mall and challenging her belief that she will become trapped with no escape

D- A CBT therapist would see the causes of disorder as stemming from the interactions between thoughts and behaviors. While a CBT therapist MAY use associative learning (A). flooding (B), or medication (C) as part of therapy, only (D) mentions correcting patterns of conscious thought and is therefore most representative.

A patient who has been diagnosed with schizophrenia is prescribed a neuroleptic medication. Which of the following symptoms, if present, is this medication most likely to improve? A) Apathy B) Inability to experience pleasure C) Emotional flattening D) Disorganized speech

D- Schizophrenia typically involves both positive symptoms, which are "pathological excesses" (eg, hallucinations, delusions, disorganized speech), and negative symptoms, which are "pathological deficits" (eg, apathy, inability to experience pleasure). Many people with schizophrenia also experience psychomotor symptoms (ie, changes in muscle tone or activity), which can occur either as a symptom of schizophrenia or as a side effect of medication. Schizophrenia is often treated with neuroleptic (conventional, typical, or first-generation) or atypical (second-generation) antipsychotic medication; both are generally effective in reducing positive symptoms. However, neuroleptics may exacerbate (worsen) negative symptoms through sedation or cognitive dulling, and they carry the risk of movement (eg, tremors) and other side effects. Atypical antipsychotics pose less risk of many of these side effects and in some cases may improve negative symptoms. A neuroleptic medication would be most likely to effectively treat (improve) disorganized speech, which is a positive symptom of schizophrenia.

A researcher hypothesizes that there is a positive correlation between symptom severity of post-traumatic stress disorder (PTSD) and external locus of control. Which hypothetical finding would support this hypothesis? A) A more distorted and unstable self-image is associated with a stronger belief that the outcomes of events are controlled by one's own actions. B) A more distorted and unstable self-image is associated with a stronger belief that the outcomes of events are due to luck, fate, or powerful others. C) More severe hyperarousal is associated with a stronger belief that the outcomes of events are controlled by one's own actions. D) More severe hyperarousal is associated with a stronger belief that the outcomes of events are due to luck, fate, or powerful others.

D. A PTSD diagnosis requires evidence of several types of symptoms, including hyperarousal (eg, exaggerated startle response, insomnia); intrusive symptoms (eg, nightmares, flashbacks); avoiding reminders of the trauma; and negative thoughts and moods. The researcher's hypothesis would be supported by the finding that more severe hyperarousal (a symptom of PTSD) is associated with a stronger belief that the outcomes of events are due to luck, fate, or powerful others (external LOC). Locus of control (LOC) refers to an individual's beliefs about who or what controls the outcome of a situation: Either a person has the ability to influence outcomes of events (internal LOC), or outcomes are determined by luck, fate, or powerful others (external LOC). Post-traumatic stress disorder (PTSD) can arise after exposure to a traumatic event and is characterized by hyperarousal, intrusive symptoms, avoidance behaviors, and negative thoughts/mood.

A researcher recruited 20 individuals with complete spinal cord injuries at cervical vertebrae C3 or C4 (participants had no sensory or motor function below the site of injury) and 20 matched controls. Participants were presented with images designed to evoke positive, negative, or no emotion. If the results indicate no significant difference between participants with spinal cord injuries and control participants in the emotional response to the images, this finding would: A) Contradict the James-Lange theory. B) Support the Schachter-Singer theory. C) Contradict the Cannon-Bard theory. D) Support the theory of universal emotions

Individuals with cervical spinal cord injury (SCI) do not receive sensory information from the body. According to the James-Lange theory, such individuals are unable to experience emotion because sensory information (eg, increased heart rate) from the body cannot reach the brain; as such, this would predict that only control participants in the present scenario would be able to experience emotion. Therefore, a finding that there is no significant difference between the emotional responses of controls and those with complete SCI contradicts the James-Lange theory by showing that controls and affected individuals display similar emotion.

Reinforcers & Punishment

Primary reinforcers fulfill a biological urge and are naturally rewarding. Examples of primary reinforcers are food, comforting physical contact, and praise. Secondary reinforcers are not inherently rewarding but are conditioned to be desirable through associative learning. For example, if a child brings home good grades, the parent may reinforce this behavior with praise or candy (primary reinforcers) so the child feels happy. The child then associates feeling good with good grades, resulting in grades becoming a secondary reinforcer. Primary punishers are naturally undesirable and cause negative emotions such as pain or fear. Examples of primary punishers include an electric shock or a loud noise. Secondary punishers are not inherently negative but are conditioned to be undesirable through associative learning. For example, if a child brings home bad grades, the parent may punish this behavior with a scolding (primary punisher) so the child feels bad. The child then associates feeling bad with bad grades, resulting in grades becoming a secondary punisher.

Prior to assimilation, ethnic groups migrating from a less industrialized culture to a more industrialized culture generally compare to their new culture in which of the following ways? A) Higher fertility, higher mortality B) Higher fertility, lower mortality C) Lower fertility, higher mortality D) Lower fertility, lower mortality

A - Ethnic groups emigrating to a more industrialized culture typically have larger families, but shorter lifespans initially.

If this study is repeated using the same procedure with a different set of subjects and produces similar results, this suggests that the study is: A) Reliable. B) Valid. C) Generalizable. D) Neutral.

A - Reliability refers to the consistency of an experiment or measure. Reliable measures produce similar results every time. For example, two official MCATs taken by the same person on consecutive days should produce similar scores. If the scores differ dramatically, this would be considered an unreliable test.

If a psychology were to analyze the data and determine that the repeated actions of the non-DRD4 thrill seekers were based on positive responses from peers, they would be adopting which personality theory perspective? A) Behavioralist perspective B) Biological perspective C) Psychoanalytic perspective D) Trait perspective

A - The psychologist is using positive reinforcement as the basis for the behavior of those without the gene variant. This determination would conform to (A), because behaviorist theory includes the idea that behavior is based upon reinforcement and punishment. (B) is the opposite, and applies to the DRD4 group, (C) would revolve around the unconscious desires of the thrill-seeker, and is not relevant, (D) is more about internal personality traits, and the psychologist is using external factors in their explanation.

Social epidemiology

A subfield focusing on the social factors that influence the health of an individual or population. Social factors can include larger social structures, such as a society at war, and more immediate factors, such as neighborhood safety. Many social factors can negatively impact health, including income and education disparities, unemployment, and food quality/availability. The study of how certain PD individuals catalyze changes to reduce malnutrition in Vietnamese villages is most applicable to social epidemiology.

A feminist theorist would be most interested in which of the following? A) Equalizing pay between females in pink-collar and blue-collar occupations B) Removing barriers to entry in all occupations for females C) Creating more access for males in pink-collar occupations D) Balancing the percentage of females and males within each occupation

At the macro-level, feminist theory considers how social structures maintain gender inequality. For example, workplace practices that discriminate against women (eg, less pay, fewer promotions) result in the underrepresentation of women in certain fields (eg, surgery). At the micro-level, feminist theory considers how one-on-one interactions also maintain gender inequality by objectifying or devaluing females/femininity. For example, patients often assume a female surgeon is a nurse.

Which of the following brain structures is most critical to the experience of emotion? A) Prefrontal cortex B) Hypothalamus C) Hippocampus D) Posterior pituitary

B - The hypothalamus is responsible primarily for the physiological component of emotion, such as changes in heart or respiration rate. The James-Lange theory of emotion (second paragraph) suggests that specific physiological reactions produce specific emotions. For example, if experiencing increased heart rate and sweating, one will experience fear. Therefore, the hypothalamus is most critical to the experience of emotion according to the James-Lange theory.

The monoamine theory states that: A) High levels of dopamine contribute to depression B) Low levels of dopamine contribute to depression C) High levels of thyroid hormone contribute to anxiety D) Low levels of thyroid hormone contribute to anxiety

B - The monoamine/catecholamine theory of depression states that too much dopamine leads to mania, while too little leads to depression. Thyroid hormone is responsible for regulating metabolism and anxiety-like symptoms (choices C and D are incorrect).

The hemisphere in which Wernicke's area is located is most associated with which of the following cognitive functions? A) Right-sided odor perception B) Right-hand control C) Spatial awareness D) Face recognition

B - Wernicke's area is located in the left side of the temporal lobe and thus the left hemisphere of the brain. The left hemisphere controls the right side of the body (choice B). While afferent sensory signals usually cross to the opposite hemisphere of the brain to undergo processing, olfaction is an exception (choice A is incorrect). Spatial awareness and emotional intelligence are functions associated with the right hemisphere (choices C and D are incorrect).

Normative social influence is most likely to arise in the group: A) Containing several members who have expertise regarding cancer. B) Composed of all females from the same university. C) With an assigned group spokesperson. D) Whose members are the most disparate in socioeconomic status.

B- Normative social influence describes when an individual conforms to fit in or avoid rejection by others. For example, a college freshman notices that everyone on campus wears yellow wristbands, so he wears one to fit in. Normative social influence would most likely arise in a group composed of females from the same university because individuals tend to want to fit in with group members with whom they identify.

When the sole of a 3-year-old girl's foot is touched with a metal prick, her toes curl upward and extend. Which of the following conclusions is most likely to be true? A) The girl exhibits a positive response; this should be considered normal B) The girl exhibits a negative response; this should be considered normal C) The girl exhibits a positive response; this should be considered abnormal D) The girl exhibits a negative response; this should be considered abnormal

C - The (positive) Babinski reflex occurs when the toes of infants up to 2 years of age automatically extend or splay outward when the sole of the foot is stimulated with a sharp object (choices B and D are incorrect). The patient described here is outside the age range at which this response should be present; thus, it is an abnormal response, and neurological damage should be considered (choice C is correct).

Researchers noticed that participants were more likely to erroneously report hearing a tone when viewing a picture of an object that produces a tone-like sound, such as a telephone or a bell. What type of processing best explains why this mistake might occur? A) Bottom-up processing B) Parallel processing C) Serial processing D) Top-down processing

D - Top-down processing is guided by ideas, beliefs, and expectations from prior experience. Bottom-up processing is guided by sensory input.

Socialization

Describes the lifelong process of learning the norms, values, and behaviors of one's own society. The process of learning through social interaction with others is the foundation of socialization. Although there are many more, the major agents of socialization include family, friends/peers, school/workplace, and mass media. Family and friends are considered the most important agents of socialization because these relationships are formed early in life and are generally long-lasting and intimate. Interaction with family and friends provides important information about societal norms and values, which tend to be internalized.

Educational stratification

Educational stratification refers to the mechanisms that produce inequality in educational access (eg, schools available to students) and outcomes (eg, graduation rates, college matriculation) in society. Students are stratified in the education system as a result of social characteristics, such as parent's income or influence. Educational stratification occurs on multiple levels. For instance, students with high social standing (eg, wealth) have many options (eg, well-funded local public schools, private schools), whereas students with low social standing have fewer options (ie, limited access to quality schools and teachers). The experts' suggestion in the final paragraph that the education system perpetuates income disparities through the unequal distribution of resources (ie, funding, quality teachers) draws most directly from the concept of educational stratification.

Neuroplasticity

Neuroplasticity can occur on the synaptic or structural level. Synaptic plasticity results from changes in the firing rate of the presynaptic neuron, which alters the amount of neurotransmitter released into the synaptic cleft and the number of receptors on the postsynaptic target. Synaptic changes are associated with both immediate and more delayed potentiation or depression. At the structural level, sprouting (increased connections between neurons), rerouting (new connections between neurons), and pruning (decreased connections between neurons) contribute to structural plasticity. Structural plasticity does not happen quickly, so it is not responsible for immediate changes

A social psychologist employs a confederate to pretend to have a heart attack in a crowded train station, then measures how long it takes for the confederate to receive help. The study procedure is repeated over multiple trials throughout the day. This experiment is most likely evaluating which phenomenon and with what independent variable, respectively? A) Bystander effect; amount of time until someone attempts to help the confederate B) Bystander effect; number of onlookers C) Hawthorne effect; amount of time until someone attempts to help the confederate D) Hawthorne effect; number of onlookers

The bystander effect occurs when an individual in need is less likely to receive help when more people are present. This study was designed to evaluate the bystander effect. The researcher likely hypothesized that a greater number of onlookers present (independent variable) in the train station would result in a longer amount of time until someone attempts to help the confederate (dependent variable).

Sleep waves

The deepest of non-REM sleep, slow-wave sleep (stages 3 and 4) is characterized by delta waves, which demonstrate the lowest frequency and highest amplitude observed during the sleep cycle. (Choice A) Alpha and beta waves are high-frequency, low-amplitude waves characteristic of waking states. Beta waves have the highest frequency and are characteristic of awake, alert states. Alpha waves have more regularity and are characteristic of awake, relaxed states. (Choices B and D) Theta waves are characteristic of stage 1 sleep ("light sleep"). During stage 2 sleep, theta waves still predominate but are interrupted by occasional sleep spindles (bursts in frequency) and K-complexes (increases in wavelength).

Theories of Identiy

The identity development theory by James Marcia evaluates the psychological progress of individuals based on their level of commitment and degree of exploration, yielding four identity statuses. Identity diffusion (low commitment, low exploration): People at this level lack direction, have not explored options, and have not committed to a particular career path or future. Identity foreclosure (high commitment, low exploration): People at this level have accepted an identity that they have been assigned (typically by a parent or authority figure) without contemplation or exploration. Identity moratorium (low commitment, high exploration): People at this level are still trying new activities and thinking about a career path, but have not yet arrived at a decision. Identity achievement (high commitment, high exploration): People at this level have explored their options and typically feel confident about who they are and what they want to do in the future.

Theories of Human Development

This study defines resilience as the ability to prevail despite threats to adequate psychosocial development. Therefore, this study appears to be basing human development on Erik Erikson's psychosocial theory. There are many different theories focusing on various aspects of human development. Psychoanalytic theories, like those of Freud and Erikson, focus on personality development. Erikson's theory focuses on how personality is shaped by social interaction throughout a lifetime.

Language and cognition

Young-Helmholtz theory, also known as the trichromatic theory, states that all the colors we see are the result of the combined activity of three types of photoreceptors: Those that respond to short (blue), medium (green), and long (red) wavelengths of light. The Sapir-Whorf hypothesis, also known as linguistic relativity, posits that language influences our perception and cognition. For example, if a person's native language does not have separate names for the colors blue and indigo (instead referring to both as "blue"), that person may have difficulty discriminating blue from indigo in isolation but could learn to differentiate them with practice.

James-Lange theory

argues that our body's physiological response to a stimulus, in addition to our subsequent perception of that response, is what makes us feel emotion.


Ensembles d'études connexes

Accounting Ch.12 Securities Classified "Trading" debt or FVTNI equity

View Set

Chapter 7.1 civil rights and 7.2 and 7.3

View Set

Womens Health: Combined Oral Contraceptives (Case 2)

View Set